Say whaaa?? How is B the answer? Forum

Prepare for the LSAT or discuss it with others in this forum.
Post Reply
yeahyeah2121

New
Posts: 98
Joined: Thu Sep 24, 2009 7:39 pm

Say whaaa?? How is B the answer?

Post by yeahyeah2121 » Fri Oct 01, 2010 11:04 pm

.
Last edited by yeahyeah2121 on Fri Oct 01, 2010 11:08 pm, edited 1 time in total.

User avatar
gdane

Diamond
Posts: 14023
Joined: Sat Sep 26, 2009 2:41 pm

Re: Say whaaa?? How is B the answer?

Post by gdane » Fri Oct 01, 2010 11:05 pm

Dont post LSAT questions and their answers.

Also, what test is this from?

yeahyeah2121

New
Posts: 98
Joined: Thu Sep 24, 2009 7:39 pm

Re: Say whaaa?? How is B the answer?

Post by yeahyeah2121 » Fri Oct 01, 2010 11:08 pm

PT 35 (I believe?) June 2001, LR #2, Question 19.

User avatar
gdane

Diamond
Posts: 14023
Joined: Sat Sep 26, 2009 2:41 pm

Re: Say whaaa?? How is B the answer?

Post by gdane » Fri Oct 01, 2010 11:10 pm

If the retina is dependant on the outside environment, specifically as it gets hotte, then hot weather can affect the stuff the eye does. Get it?

yeahyeah2121

New
Posts: 98
Joined: Thu Sep 24, 2009 7:39 pm

Re: Say whaaa?? How is B the answer?

Post by yeahyeah2121 » Fri Oct 01, 2010 11:12 pm

I just don't see how on earth B could be the answer. Truly, I'm mind boggled and I'm usually pretty good at LR.

Want to continue reading?

Register now to search topics and post comments!

Absolutely FREE!


User avatar
gdane

Diamond
Posts: 14023
Joined: Sat Sep 26, 2009 2:41 pm

Re: Say whaaa?? How is B the answer?

Post by gdane » Fri Oct 01, 2010 11:14 pm

You have to remember that this is a must be true question. If the activity of the retina is proportional to the outside temperature, then if it were to get hotter it would cause the eye to do a bunch of crazy stuff.

yeahyeah2121

New
Posts: 98
Joined: Thu Sep 24, 2009 7:39 pm

Re: Say whaaa?? How is B the answer?

Post by yeahyeah2121 » Fri Oct 01, 2010 11:19 pm

Oh wow. I completely misinterpreted the last sentence. Whooopssss. Still, I think it's a bad question. It makes you assume that retina temperature is directly related to external temperature. Not a leap I was willing to make but yeah I guess it's the "most supported" of the 5. Thanks.

Want to continue reading?

Register for access!

Did I mention it was FREE ?


Post Reply

Return to “LSAT Prep and Discussion Forum”